What a chain! 4

A chain of uniform mass density λ \lambda and mass M M and length L L is hanging from ceiling through a string. The other end of the chain is held at rest at the position shown in the figure. At time t = 0 t=0 the held part of chain is left to fall under gravity. Find the tension in the string as a function of time ( t ) (t) (neglecting friction). Your answer can be represented as

T ( t ) = M g a + b M g 2 t 2 c L T(t) = \frac{Mg}{a} + \frac{b Mg^2t^2}{c L}

where a a , b b , and c c are positive coprime integers. Enter your answer as a + b + c a + b + c .

Note : Figures are not to scale.


This is inspired by Depanshu's Chain please don't fall! and is a part of my set Aniket's Mechanics Challenges .


The answer is 9.

This section requires Javascript.
You are seeing this because something didn't load right. We suggest you, (a) try refreshing the page, (b) enabling javascript if it is disabled on your browser and, finally, (c) loading the non-javascript version of this page . We're sorry about the hassle.

2 solutions

Md Zuhair
Aug 12, 2017

First of all lets find out the Tension as a function of distance falled(y) or T ( y ) T(y) .

So We can see that L + y 2 λ = m \dfrac{L+y}{2} \lambda =m , where m=mass of the left portion.

d m d t = λ 2 d y d t \dfrac{dm}{dt} = \dfrac{\lambda}{2} \dfrac{dy}{dt}

F t h r u s t = λ v 2 \implies F_{thrust} = \dfrac{\lambda v}{2} and v = 2 g y v=\sqrt{2gy} .

Now the force due to the mass of the rope,

F w e i g h t = T m g F_{weight}= T- mg .

Also F w e i g h t F t h r u s t = m a F_{weight}-F_{thrust}=ma ....

Hence F w e i g h t F t h r u s t = m × 0 F_{weight}-F_{thrust}= m \times 0 as there is no accleration of the left chain.

F w e i g h t = F t h r u s t F w e i g h t = λ v 2 = λ g y F_{weight} = F_{thrust} \implies F_{weight}=\dfrac{\lambda v}{2} = \lambda gy

So putting in the equation with Tension we get

T ( y ) = λ g 2 ( L + 3 y ) T(y)\quad =\quad \dfrac { \lambda g }{ 2 } (L+ 3y)

Now we know v 2 = 2 g y v^2=2gy and v = g t v=gt

So y = 2 g t y=\dfrac{2g}{t}

Again T ( t ) = λ g 2 ( L + 3 2 g t ) M g 2 + 3 M g 2 t 2 4 L T(t)\quad =\quad \dfrac { \lambda g }{ 2 } (L+ 3\dfrac{2g}{t}) \implies \dfrac{Mg}{2}+\dfrac{3Mg^2t^2}{4L}

So a + b + c = 2 + 3 + 4 = 9 a+b+c=2+3+4=9

@Md Zuhair we cannot write v=sqrt (2gy)..... it should be v=sqrt [g (2yl-y^2)÷(l-y)] ...So the answer is wrong

Aaron Jerry Ninan - 3 years, 7 months ago

Log in to reply

@Aaron Jerry Ninan Even I got the same v as you have mentioned...Did you use energy conservation? (I used it)..I don't know what is the reason why energy is not being conserved ... @Md Zuhair @Thomas Jacob

Ankit Kumar Jain - 3 years, 3 months ago

Log in to reply

Isnt it working with energy conservations? Um.. let me think, Will tell you in hangout when i get to know the reason

Md Zuhair - 3 years, 3 months ago

Log in to reply

@Md Zuhair There are inelastic collisions between the various links of the chain, as one link receives the velocity of the other ones

Suhas Sheikh - 3 years, 2 months ago

I got velocity using energy conservation.....the expression for velocity is the one that I posted and sqrt (2gy) is wrong.....Also I saw the same question to calc velocity in Dc pandey and the ans given is v=sqrt [g (2yl-y^2)÷(l-y)] . @Ankit Kumar Jain @Md Zuhair

Aaron Jerry Ninan - 3 years, 3 months ago

Log in to reply

@Aaron Jerry Ninan Which chapter was it in?

Ankit Kumar Jain - 3 years, 3 months ago

Log in to reply

@Ankit Kumar Jain It was in a old Dc pandey problem book....not there in the newer version

Aaron Jerry Ninan - 3 years, 3 months ago

@Aaron Jerry Ninan But at the same time, what is wrong with taking the motion to be free fall?

Ankit Kumar Jain - 3 years, 3 months ago

Log in to reply

@Ankit Kumar Jain @Ankit Kumar Jain ....see the first answer in the given link https://physics.stackexchange.com/questions/278500/falling-chain-fixed-at-one-end-force-at-the-hinge

Aaron Jerry Ninan - 3 years, 3 months ago

Well this approach is incorrect according to INPhO 2019 .Should use energy conservation

saptarshi dasgupta - 2 years, 4 months ago

Log in to reply

According to INPhO 2019, Chain was elastic also... There is a huge difference there i guess?

Md Zuhair - 2 years, 4 months ago

the correct method is in fact energy conservation

Ankit Kumar Jain - 2 years, 3 months ago

aaron had said that last year itself , that is something great on his part..

Ankit Kumar Jain - 2 years, 3 months ago

The tension rises as a function of time as mass of the hanging chain ( effective weight ) increases

0 pending reports

×

Problem Loading...

Note Loading...

Set Loading...